4
$\begingroup$

Let us define the density of subset $A\subset\omega$ : $$\rho(A)=\lim_{n\to\infty}\frac{|A\cap n|}{n}$$ if the limit exists. Let $\mathcal{F_1}=\{A\subset\omega~|~\rho(A)=1\}$. $\mathcal{F_1}$ is the filter and for the Frechet filter we have $\mathcal{N}\subset\mathcal{F_1}$. For arbitrary selective ultrafilter $\mathcal{U}$ let $\mathcal{F}=\mathcal{F_1}\cap\mathcal{U}$.

Question: is there exists a bijection $\varphi:\omega\times\omega\to\omega$ such that $$ \varphi(\mathcal{F}\otimes\mathcal{F})\subset\mathcal{U} $$

$\endgroup$
6
  • $\begingroup$ I don't understand the reason of downvote. Can anyone explain what is the problem with question? $\endgroup$
    – ar.grig
    Mar 21, 2019 at 14:15
  • 2
    $\begingroup$ I conjecture there's no reason for the downvote. Someone has also downvoted my answer here, my answers to your earlier questions, and my answer to a question about the possible simultaneous existence of P-points and Q-points. In no case was a reason for the downvote given. The simplest explanation seems to be that somebody (I hope just one person) doesn't like ultrafilters. $\endgroup$ Mar 24, 2019 at 19:35
  • $\begingroup$ May be my bad English is the problem. The other reason is the questions quality. I am newbie to ultrafilters and sets theory but the questions are coming from research in other sphere. $\endgroup$
    – ar.grig
    Mar 25, 2019 at 7:03
  • 2
    $\begingroup$ I don't think English is the problem. I just got another unexplained downvote on another ultrafilter question, mathoverflow.net/questions/326274 . $\endgroup$ Mar 25, 2019 at 17:30
  • 2
    $\begingroup$ I have been getting many unexplained downvotes and delete votes recently too from people who don't like set theory and similar areas. mathoverflow.net/a/320749/22277 mathoverflow.net/q/321898/22277 mathoverflow.net/q/321894/22277 mathoverflow.net/q/326224/22277 mathoverflow.net/q/321504/22277 mathoverflow.net/q/326024/22277 $\endgroup$ Mar 26, 2019 at 3:32

1 Answer 1

4
$\begingroup$

No, there do not exist such selective $\mathcal U$ and bijection $\varphi$. Since selectivity is preserved by bijections and since the Fréchet filter $\mathcal N$ is included in $\mathcal F_I\cap\mathcal U$, it suffices to show that no selective ultrafilter $\mathcal U$ on $\omega\times\omega$ includes $\mathcal N\otimes\mathcal N$.

Suppose, toward a contradiction, that we had a selective ultrafilter $\mathcal U\supseteq\mathcal N\otimes\mathcal N$. Partition $\omega\times\omega$ into the columns $\{n\}\times\omega$. Such a column cannot be in $\mathcal U$ because its complement $(\omega-\{n\})\times\omega$ is in $\mathcal N\otimes\mathcal N$ and therefore in $\mathcal U$. So, by selectivity, $\mathcal U$ contains a set $A$ that meets each column at most once. But the complement $(\omega\times\omega)-A$ of this $A$ is in $\mathcal N\otimes\mathcal N$, because it contains a cofinite (in fact at least co-singleton) part of every column. So $(\omega\times\omega)-A\in\mathcal N\otimes\mathcal N\subseteq\mathcal U$, which, together with $A\in\mathcal U$, contradicts the fact that $\mathcal U$ is a filter.

$\endgroup$
5
  • 1
    $\begingroup$ The argument is easily modified to show that no P-point includes $\mathcal N\otimes\mathcal N$. And in this form, there's a converse: An nonprincipal ultrafilter on a countable set is a P-point if and only if no isomorphic copy of it on $\omega^2$ extends $\mathcal N\otimes\mathcal N$ $\endgroup$ Mar 24, 2019 at 19:31
  • $\begingroup$ Thus none of Katetov's idempotent filters can be subfilter of a selective ultrafilter. What about existing of non-principal subfilter $\mathcal{F}$ of selective ultrafilter $\mathcal{U}$ with the property $\varphi(\mathcal{F}\otimes\mathcal{F})\subset\mathcal{U}$? $\endgroup$
    – ar.grig
    Mar 25, 2019 at 7:24
  • 1
    $\begingroup$ @ar.grig I assume you still want $\varphi$ to be a bijection (if it's merely a function, take it to be either projection $\omega^2\to\omega$ and take $\mathcal F=\mathcal U$) and that "non-principal" means containing all cofinite sets (there are at least two meanings of "non-principal" for filters; they agree for ultrafilters but not in general). Then the answer is no. As in my earlier answer, $\varphi$ is irrelevant, $\mathcal F$ extends the Fréchet filter $\mathcal N$, and no selective ultrafilter extends $\mathcal N\otimes\mathcal N$. $\endgroup$ Mar 25, 2019 at 12:32
  • $\begingroup$ I want $\varphi$ to be bijection and non-principal filter in meaning of empty intersection of filter. $\endgroup$
    – ar.grig
    Mar 25, 2019 at 13:21
  • 1
    $\begingroup$ @ar.grig Then my previous comment provides a negative answer to your next-to-previous comment. A filter whose intersection is empty must contain all cofinite sets. $\endgroup$ Mar 25, 2019 at 14:10

Your Answer

By clicking “Post Your Answer”, you agree to our terms of service and acknowledge you have read our privacy policy.

Not the answer you're looking for? Browse other questions tagged or ask your own question.